Logic Games Random Mix - - Question 18
Which one of the following could be true?
Replies
Irina February 24, 2020
@hatemz,This is a linear game that requires us to rank 8 chips - F G H J K L M O from the fastest (1) to the slowest (8).
G/F__ __ __ __ __ __ __
1 2 3 4 5 6 7 8
~M
The following rules apply:
(1) Either G/F is ranked first.
(2) M is not the slowest
(3) H is faster than J and exactly one chip is between them.
H_J
(4) K is faster than L and exactly two chips are between them.
K__ __ L
(5) O is slower than both J & L
J
> O
L
This rule tells us that there must be an overlap between H_J group and K __ __ L, otherwise there are 8 chips already - 3 for H_J, 4 for K __ __ L, and O without counting the first chip, which is F or G. Since we only have 8 chips, there must be an overlap. This leaves us with four possible scenarios:
F/G F/G/M K F/G/M H L J O
F/G F/G/M H K J F/G/M L O
F/G K __ H L J __ F/G/O
F/G H K J __ L __ F/G/O
The question asks us which of the following could be true:
(A) is incorrect because M cannot be the slowest per the rules
(B) is correct. In scenario 3 above, G could be ranked 5th and O is ranked 8th.
(C) is incorrect. It is not possible in any scenario.
(D) is incorrect because even though K could be ranked second, H cannot be ranked third because it would leave no space for L - K H X J/L - in this scenario both J and L have to be in the same slot and there are no ties per the rules, hence it is impossible.
(E) is incorrect because even though M could be ranked seventh, L cannot be last because O is slower and is ranked lower than L per the rules.
Does this make sense?
Let me know if you have any other questions.
AneeshU August 24, 2022
For anyone else struggling with this, I think there might be a typo in this answer^. B is based on scenario 4, not 3.